LSAT and Law School Admissions Forum

Get expert LSAT preparation and law school admissions advice from PowerScore Test Preparation.

 Administrator
PowerScore Staff
  • PowerScore Staff
  • Posts: 8950
  • Joined: Feb 02, 2011
|
#22767
Complete Question Explanation

Must Be True—SN. The correct answer choice is (A)

In this stimulus we are provided a fact set which can be diagrammed as follows:

Every early 20th century political philosopher who was socialist or communist was influenced by Luxemburg. In other words, if you were one of those two types, then you were a Luxemburg follower:

  • Communist 20th C. pol. philosopher

    ..... ..... ..... Or ..... ..... ..... :arrow: ..... Influenced by Luxemburg

    Socialist 20th C. pol. philosopher

Further, if you were influenced by Luxemburg, you did not support a totalitarian state. This can be added to the original diagram as follows:

Conditional Diagram:

  • Communist 20th C. pol. philosopher

    ..... ..... ..... Or ..... ..... ..... :arrow: ..... Influenced by Luxemburg ..... :arrow: ..... non-totalitarian

    Socialist 20th C. pol. philosopher

Contrapositive:

  • ..... ..... ..... ..... ..... ..... ..... ..... ..... ..... communist 20th C...

    Support totalitarian :arrow: influenced by Luxemburg :arrow: ..... AND

    ..... ..... ..... ..... ..... ..... ..... ..... ..... ..... socialist 20th C...

Since the stimulus is followed by a Must Be True question, it will likely be answered by one of the two diagrams above.

Answer choice (A): This is the correct answer choice, as it is an inference which can be drawn based on the conditional diagram from our discussion above. If every early 20th century Socialist political philosopher was influenced by Luxemburg, and such influence meant that you were not a supporter of a totalitarian state, then the inference in this answer choice is certainly valid. Note: it is easy to miss the reference to "socialist" in this answer and think it refers to all philosophers; it does not. It refers specifically to socialists.

Answer choice (B): This incorrect answer choice is a mistaken reversal of the last portion of the first conditional diagram above. Since we can not go against the direction of the arrows in such a diagram, this is not a valid inference.

Answer choice (C): This answer choice gives us something close to a valid contrapositive, but not quite: Let's consider the relevant portion of the contrapositive diagrammed above:

  • ..... ..... ..... ..... ..... ..... ..... communist 20th C...

    influenced by Luxemburg ..... :arrow: ..... ..... AND

    ..... ..... ..... ..... ..... ..... ..... socialist 20th C...

So, if you were not influenced by Luxemburg, then you were neither a communist nor socialist early 20th century political philosopher.

But this incorrect answer choice asserts that if you weren't Luxemburg herself, then you didn't influence both communist and socialist early 20th century political philosophers.

Answer choice (D): This answer choice presents a strange hybrid mistake which is unsupported by either diagram above, so it must be incorrect.

Answer choice (E): Like answer choice (D) above, this choice does not make one of the classic mistakes (negation, reversal), but is unsupported by the information provided in the stimulus, so it is incorrect.
 rachue
  • Posts: 140
  • Joined: Jun 22, 2011
|
#706
Hi. I have a question about a LR question from Lesson 2 (page 2-67). This is the question:

[Question Removed by Admin -- LSAT questions cannot be posted per LSAC question disclosure rules. Just referencing the Homework Lesson and Question Number as you did in the post title gives us all the info we need! ]

I can narrow the answers down to A and D.


When I do the diagramming how Powerscore teaches, I would choose A based on that diagram. However, I can't see the reasoning as to why D must be false.

Can someone please explain to me why D must absolutely be false?
 Jon Denning
PowerScore Staff
  • PowerScore Staff
  • Posts: 907
  • Joined: Apr 11, 2011
|
#755
Hey rachue - thanks for the question. I don't think the idea behind D is that it CANNOT be true (i.e. that it must be false), but rather that it doesn't HAVE to be true (i.e. it could be false). Since we're looking to find an answer choice that is absolutely proven true by the statements in the stimulus, we cannot choose D because we simply don't know if it's true. That's enough to throw out an answer in a must be true question.

Note how A is proven true - every correct answer in must be true will be completely supported by the information in the stimulus.

Hope that helps!

JD
 rachue
  • Posts: 140
  • Joined: Jun 22, 2011
|
#756
That does help a lot! It all just hit me. I think I needed someone else to explain it to me so I could see it from another angle. Thanks! :D
 ellenb
  • Posts: 260
  • Joined: Oct 22, 2012
|
#8836
Dear Powerscore,

I have read the explanation for this question, but I am still confused with why C is wrong?

Thanks in advance

Ellen
 Justin Eleff
PowerScore Staff
  • PowerScore Staff
  • Posts: 19
  • Joined: Jul 27, 2012
|
#8919
OK. So the question says everyone who was either a Political Philosopher: Socialist or a Political Philosopher: Communist was influenced by Rosa L. Which we might scribble out as PPS or PPC -> iRL. And we know that having been influenced by RL meant not advocating a totalitarian state, because that's what the second sentence of the stimulus says exactly. So we can expand our scribbling to PPS or PPC -> iRL -> NOT(aTS). And by virtue of that, we know for sure that answer choice (A) is correct, because we know that PPS leads inevitably to NOT(aTS), which is to say, to NOT ADVOCATING A TOTALITARIAN STATE. Now, we also know it leads inevitably to iRL, to having been INFLUENCED BY ROSA L. But answer choice (C) takes that too far. We know that all of these people were indeed influenced by Rosa L., but we have no way of concluding that she was the ONLY person to influence all of them. The fact that she DID influence people does not have any bearing on whether others DID or DID NOT. So the use of the word "only" in (C) takes that answer choice outside the four corners of the stimulus. We just don't know enough to pick that as something that must be true.
 ellenb
  • Posts: 260
  • Joined: Oct 22, 2012
|
#11867
Dear Powerscore,

I got this answer correct and I understood the diagram.

I just diagramed it a bit differently:

Political Philosopher-->Socialist or Communist--->infuenced by Luxemburg-->not totalitarian
(early 12 Century)

and in the explanations they have:

Political Philosopher Socialist

or --->Influenced by Luxemburg-->not totalitarian

Political Philosopher Communist

1)Which way is correct? because technically the answer choice A supports my diagram more, since "no early 12 century political philosopher supported a totalitarian state"
vs. the diagram in the explanations has to statements connected by or and one is socialist and the other is communist.

2) How would you diagram answer choice C? and why it is wrong?
the diagram seems very similar to the one we had in the stimulus.
political philosopher socialist or political philosopher communist--> influenced by Luxemburg
(ONLY)
3) How would you diagram answer choice E, it seems like a mistake reversal, however they state in the explanations that it is neither.

Not totalitarian state advocated--> was a socialist or a communist

Thanks in advance!

Ellen
User avatar
 KelseyWoods
PowerScore Staff
  • PowerScore Staff
  • Posts: 1079
  • Joined: Jun 26, 2013
|
#11892
Hi Ellen,

Thanks for your questions!

1) The diagram in the explanations is correct. The first part of your diagram reads "All political philosophers are socialists or communists," but that's not quite what the stimulus said. The stimulus refers to "Every political philosopher who was either a socialist or a communist." There could be many other types of political philosophers (capitalists, anarchists, etc.) so our author is not saying that ALL political philosophers are socialists or communists. The author is instead referring just to the political philosophers that are socialist or communist.

The diagram in the explanations also supports answer choice (A). When you have two conditions at the beginning of a conditional statement, if you meet either one then you can follow the arrow to the next condition. So, based on that diagram, if you are a political philosopher who is a socialist, I know you were influenced by Luxemburg and therefore did not advocate a totalitarian state. Likewise, if you are a political philosopher who is a communist, you were also influenced by Luxemburg and did not advocate a totalitarian state. Being either a socialist political philosopher or a communist political philosopher is enough to tell me that you were influenced by Luxemburg and therefore did not advocate a totalitarian state.

2) Answer choice (C) is incorrect because the stimulus says that all of the socialist and capitalist political philosophers were influenced by Rosa Luxemborg but it doesn't say that she is the ONLY person to influence them. I wouldn't diagram that one because it's easier to get rid of it just by thinking through the logic. But a correct diagram for it would be something like:
Person who influenced political philosophers who were socialist or communist -----> Luxemburg

3) Answer choice (E) is not quite a mistaken reversal because it refers to "every political philosopher who did not advocate a totalitarian state." A true mistaken reversal would refer to everyone who did not advocate a totalitarian state, not just the political philosophers.

Hope this helps!

Best,
Kelsey
 scharles35
  • Posts: 15
  • Joined: Apr 19, 2014
|
#15103
I see why (A) is correct i understand thanks to your explanation but can you explain why (D) is wrong please ?
User avatar
 KelseyWoods
PowerScore Staff
  • PowerScore Staff
  • Posts: 1079
  • Joined: Jun 26, 2013
|
#15147
Hi scharles35!

Answer choice (D) is wrong because it is not supported by our diagram of the conditional relationships in the stimulus.

If we use Justin's diagram for the stimulus, it looks like this:

PPS or PPC :arrow: iRL :arrow: NOT(aTS)

That's the information that we must use to prove the correct answer.

Answer choice (D) would be diagrammed like this:

iRL and NOT(PPS) :arrow: PPC

Basically, the answer choice is saying that if a political philosopher was influenced by RL and not a socialist, then that political philosopher was a communist. But that doesn't match our stimulus diagram or our contrapositive of the stimulus diagram. This answer choice rearranges the elements from the first conditional statement so that they feel familiar and tempting, but it isn't actually supported by the conditional statements we have in the stimulus.

Hope this helps!

Best,
Kelsey

Get the most out of your LSAT Prep Plus subscription.

Analyze and track your performance with our Testing and Analytics Package.